Use non-breaking space (~) between 'Figure' and the figure number.
[course.git] / latex / problems / Serway_and_Jewett_8 / problem23.62.tex
1 \begin{problem*}{23.62}
2 Four identical charged particles ($q=+10.0\U{$\mu$C}$) are located on
3 the corners of a rectangle as shown in Figure~P23.62.  The dimensions
4 of the rectangle are $L=60.0\U{cm}$ and $W=15.0\U{cm}$.
5 Calculate \Part{a} the magnitude and \Part{b} the direction of the
6 total electric force exerted on the charge at the lower left corner by
7 the other three charges.
8 % L in x direction, W in y
9 \end{problem*}
10
11 \begin{solution}
12 \Part{a}
13 Summing the electric force due to each source,
14 \begin{align}
15   \vect{F} &= kq^2 \p({
16     \frac{-\ihat}{L^2} + \frac{-\jhat}{W^2}
17     + \frac{-\frac{L}{\sqrt{L^2+W^2}}\ihat - \frac{W}{\sqrt{L^2+W^2}}\jhat}
18            {L^2+W^2}
19                       })
20     = -kq^2 \p[{  \p({\frac{1}{L^2}+\frac{L}{(L^2+W^2)^{3/2}}})\ihat
21                 + \p({\frac{1}{W^2}+\frac{W}{(L^2+W^2)^{3/2}}})\jhat}] \\
22     &= \p({-0.478\ihat - 4.05\jhat})\U{MN} \;.
23 \end{align}
24
25 The magnitude of $\vect{F}$ is therefore
26 \begin{equation}
27   |\vect{F}| = \sqrt{F_x^2 + F_y^2}
28     = \ans{4.08\U{MN}} \;.
29 \end{equation}
30
31 \Part{b}
32 Using basic trig
33 \begin{equation}
34   \theta = \arctan\p({\frac{F_y}{F_x}})
35     = 83.3\dg + 180\dg = \ans{263\dg}
36 \end{equation}
37 measured counter clockwise from the $x$ axis.
38 \end{solution}